LSAT and Law School Admissions Forum

Get expert LSAT preparation and law school admissions advice from PowerScore Test Preparation.

User avatar
 Dave Killoran
PowerScore Staff
  • PowerScore Staff
  • Posts: 5852
  • Joined: Mar 25, 2011
|
#45458
Complete Question Explanation
(The complete setup for this game can be found here: lsat/viewtopic.php?t=13934)

The correct answer choice is (C)

There are only two possible pairs that fit the numerical criterion in this question: J and L, and K and N. J and L are not one of the pairs in the answer choices, so the answer choice must be K and N. Illnesses K and N both have only one symptom, and both could have symptom F (K must have F; N could have F). Thus answer choice (C) is correct.

Get the most out of your LSAT Prep Plus subscription.

Analyze and track your performance with our Testing and Analytics Package.